LSAT - assumption question

This topic has expert replies
Master | Next Rank: 500 Posts
Posts: 165
Joined: Wed Jan 26, 2011 7:33 pm
Location: Pune, India
Thanked: 16 times
Followed by:1 members

LSAT - assumption question

by tetura84 » Sun Feb 06, 2011 10:31 am
Modern physicians often employ laboratory tests, in addition to physical examinations, in order to diagnose diseases accurately. Insurance company regulations that deny coverage for certain laboratory tests therefore decrease the quality of medical care provided to patients.
Which one of the following is an assumption that would serve to justify the conclusion above?
(A) Physical examinations and the uncovered laboratory tests together provide a more accurate diagnosis of many diseases than do physical examinations alone.
(B) Many physicians generally oppose insurance company regulations that, in order to reduce costs, limit the use of laboratory tests.
(C) Many patients who might benefit from the uncovered laboratory tests do not have any form of health insurance.
(D) There are some illnesses that experienced physicians can diagnose accurately from physicians examination alone.
(E) Laboratory tests are more costly to perform than are physical examinations.

A

I choose E.
[spoiler]My question is doesn't the assumption restate first line? assumption is supposed to be an unstated premise.[/spoiler]

Legendary Member
Posts: 1578
Joined: Sun Dec 28, 2008 1:49 am
Thanked: 82 times
Followed by:9 members
GMAT Score:720

by maihuna » Sun Feb 06, 2011 11:09 am
E is not an assumption, as cost of the test may explain why insurance companies are not covering it, but, it can not explain why it decrease the quality care provisded to patients.

A: Makes sense as it explictly saye the physical examination together with these tests are more effective.
Charged up again to beat the beast :)

User avatar
Senior | Next Rank: 100 Posts
Posts: 47
Joined: Tue Mar 02, 2010 5:47 pm
Thanked: 2 times

by gtr02 » Sun Feb 06, 2011 3:33 pm
tetura84 wrote:Modern physicians often employ laboratory tests, in addition to physical examinations, in order to diagnose diseases accurately. Insurance company regulations that deny coverage for certain laboratory tests therefore decrease the quality of medical care provided to patients.
Which one of the following is an assumption that would serve to justify the conclusion above?
(A) Physical examinations and the uncovered laboratory tests together provide a more accurate diagnosis of many diseases than do physical examinations alone.

(E) Laboratory tests are more costly to perform than are physical examinations.

My question is doesn't the assumption restate first line? assumption is supposed to be an unstated premise
Conclusion - Insurance company regulations that deny coverage for certain laboratory tests therefore decrease the quality of medical care provided to patients

Try negation method to see which hurts the conclusion more.
(A) Physical examinations and the uncovered laboratory tests together DO NOT provide a more accurate diagnosis of many diseases than do physical examinations alone.

(E) Laboratory tests are NOT more costly to perform than are physical examinations.

Master | Next Rank: 500 Posts
Posts: 165
Joined: Wed Jan 26, 2011 7:33 pm
Location: Pune, India
Thanked: 16 times
Followed by:1 members

by tetura84 » Mon Feb 07, 2011 12:07 am
guys, does not choice A just restates the premise?
Modern physicians often employ laboratory tests, in addition to physical examinations, in order to diagnose diseases accurately
AND
(A) Physical examinations and the uncovered laboratory tests together provide a more accurate diagnosis of many diseases than do physical examinations alone.

So A cannot be assumption as it is already stated. It could be right answer for must be true type questions.

E is wrong I agree, but why A is correct?

User avatar
GMAT Instructor
Posts: 1031
Joined: Thu Jul 03, 2008 1:23 pm
Location: Malibu, CA
Thanked: 716 times
Followed by:255 members
GMAT Score:750

by Brian@VeritasPrep » Mon Feb 07, 2011 3:38 pm
Hey Tetura84,

Good question, which brings up a couple things about Assumption questions:

1) The correct answer in an Assumption question isn't typically compelling, new information - because it's being assumed, it's something that should be pretty reasonable anyway given the existing premises. So it's not uncommon for the correct answer to seem altogether unexciting.

2) Probably the best way to assess an assumption is to ask whether the argument could still hold if the opposite were true. In this case, the opposite of A is:

Physical exams and lab tests together DO NOT PROVIDE a more accurate diagnosis than do physical exams alone.

That completely undermines the conclusion, which is that by refusing to cover lab tests the insurance companies are risking their patients' health. If the lab tests don't provide extra value, then the insurers aren't taking that risk.

3) Another way to look at these is one I've borrowed from David@VeritasPrep - the correct answer on an Assumption question isn't a "sword" to help further your argument, but rather a "shield" to help protect the information you already have. Here, we're not seeking out powerful new information with A, but rather looking to defend the premises as they meet the conclusion. We don't need to prove that lab tests are the most effective methods...just that they do add some value as the argument already assumes they do. The job of a correct assumption really is to protect the existing premises.


__________________________________________________________

Now, to take a look specifically at what you wrote:

Modern physicians use lab tests to diagnose diseases

is different from

Lab tests provide a more accurate diagnosis

Yes, they're similar statements, but the premise only states that one group uses these tests; the second is a declaration of fact. If you change the context, you could argue that these two statements are basically the same as those above:


-Ptolemy based his theories of sunrise and sunset on the knowledge that the sun revolved around the earth

and

-The sun did revolve around the earth

Just because someone highly-respected uses something for a particular purpose does not mean that it serves that purpose accurately! In some cases the truth is a "small" assumption, but in assumption questions a lot of times that's what the argument requires.
Brian Galvin
GMAT Instructor
Chief Academic Officer
Veritas Prep

Looking for GMAT practice questions? Try out the Veritas Prep Question Bank. Learn More.

Legendary Member
Posts: 857
Joined: Wed Aug 25, 2010 1:36 am
Thanked: 56 times
Followed by:15 members

by AIM GMAT » Sat Feb 19, 2011 6:51 am
Brian@VeritasPrep wrote:3) Another way to look at these is one I've borrowed from David@VeritasPrep - the correct answer on an Assumption question isn't a "sword" to help further your argument, but rather a "shield" to help protect the information you already have. .
Simply superb insight . Thanks Brain and David . :)
Thanks & Regards,
AIM GMAT